Soal Dewasa Seri 3 dari 5 Flashcards

1
Q

A 55-year-old man presents to the emergency department with severe, progressive shortness of breath at rest over the last 12 hours. He denies any cardiac history, including MI, CHF, and known valvular heart disease. He denies any history of hypertension, hyperlipidemia, diabetes, or a family history of premature CAD, although he smokes two packs of cigarettes daily. He denies any history of chest, back, or abdominal discomfort. A recent treadmill ECG was normal, according to the patient.
• The patient appears dyspneic after moving to the examination table.
• Pulse 120 bpm.
• BP is 250/120 mm Hg.
• Respiratory rate is 45 breaths perminute.
• There is an elevated jugular venous pulse to the angle of his jaw.
• Inspiratory crackles are noted in all lung fields.
• His heart is tachycardic and regular, an S4is present, and no murmurs are audible.
• His abdomen is thin and soft; systolic and diastolic bruits are noted.
• His extremities are warm, bilateral femoral artery bruits are noted, and the femoral pulses are palpable but diminished.
• ECG shows sinus tachycardia and borderline LV hypertrophy (LVH) by voltage criteria. No acute ST segment or T-wave changes are present.
• Chest radiograph reveals a normal mediastinum with bilateral perihilar alveolar infiltrates.
Which of the following is included in the most appropriate initial management of this patient?

A

IV furosemide, sodium nitroprusside IV infusion, and O2

How well did you know this?
1
Not at all
2
3
4
5
Perfectly
2
Q

All of the following statements regarding therapy with enalapril are true. Except:

A

Plasma renin and angiotensin-II levels increase with therapy.

How well did you know this?
1
Not at all
2
3
4
5
Perfectly
3
Q

Which of the following methods can improve patient adherence to antihypertensive therapy?

A

all of the above

How well did you know this?
1
Not at all
2
3
4
5
Perfectly
4
Q

ACE-inhibitor use is indicated in all of the following patients,except:

A

a 34-year-old woman with eclampsia who is 36 weeks pregnant

How well did you know this?
1
Not at all
2
3
4
5
Perfectly
5
Q

A 55-year-old man with type 2 DM

Choices

A

less than 130/85 mm Hg

How well did you know this?
1
Not at all
2
3
4
5
Perfectly
6
Q
A 70-year-old woman with NYHA class II CHF
Choices
A

less than 130/85 mm Hg

How well did you know this?
1
Not at all
2
3
4
5
Perfectly
7
Q

A 70-year-old woman with uncomplicated hypertension

Choices

A

less than 140/90 mm Hg

How well did you know this?
1
Not at all
2
3
4
5
Perfectly
8
Q

A 36-year-old man with type 1 DM and proteinuria greater than 1 g per 24 hours
Choices

A

less than 125/75 mm Hg

How well did you know this?
1
Not at all
2
3
4
5
Perfectly
9
Q

A 65-year-old woman with a serum creatinine of 2.0 g per dL

A

less than 130/85 mm Hg

How well did you know this?
1
Not at all
2
3
4
5
Perfectly
10
Q

Target BP for patients older than 60 years with isolated systolic hypertension is below 160/90 mm Hg.

A

F

How well did you know this?
1
Not at all
2
3
4
5
Perfectly
11
Q

A 78-year-old woman with CHF (EF, 25%), chronic AFib, gastroesophageal reflux disease, hypertension (HTN), hyperlipidemia, diabetes, and osteoporosis takes 12 different pills. At the recent senior citizen day at the local church, a nurse told her that she does not need to take digoxin because she is on amiodarone. She wants to eliminate digoxin from her medication regimen and she wants to know why you put her on it in the first place. What is your answer?

A

Digoxin reduces hospitalization.

How well did you know this?
1
Not at all
2
3
4
5
Perfectly
12
Q

A 49-year-old man is admitted with new-onset heart failure. He is diagnosed with dilated cardiomyopathy with an EF of 20%. On hospital day 1, he is diuresed and started on a regimen of furosemide, digoxin,acetylsalicylic aspirin, captopril, and simvastatin. A medical student wants to know why you did not start him on a beta-blocker. What is your explanation

A

Beta-blockers should be started in stable CHF patients.

How well did you know this?
1
Not at all
2
3
4
5
Perfectly
13
Q

The same medical student wants to know whether the patient should also be started on calcium channel blockers. What is your answer?

A

Amlodipine proved to be of small benefit in a New York Heart Association (NYHA) class III or IV patient with an EF less than 30%. This benefit was seen more in dilated cardiomyopathy patients.

How well did you know this?
1
Not at all
2
3
4
5
Perfectly
14
Q

Which of the following is true for management of acute ST segment-elevation MI?

A

Streptokinase and aspirin each have a similar effect on outcome.

How well did you know this?
1
Not at all
2
3
4
5
Perfectly
15
Q

Which of the following have beta-blockers for acute MI been shown to do?

A

all of the above

How well did you know this?
1
Not at all
2
3
4
5
Perfectly
16
Q

In diabetic patients with acute MI, the rate of mortality compared with nondiabetic patients is approximately which of the following?

A

twice as high

17
Q

ACE inhibitors have been shown to reduce mortality.

A

T

18
Q

Two weeks after a hospitalization for an anterior wall MI, a 58-year-old man returns with reports of fever and chest pain. Laboratory studies are significant for an elevated white blood cell count. An ECG reveals ST elevation in the anterior, lateral, and inferior leads. Which of the following is the correct diagnosis?

A

Dressler’s syndrome

19
Q

Diabetes raises the risk of MI the same amount as does having a history of prior MI.

A

T

20
Q

Which of the following treatments most consistently improves EF in patients who have systolic heart failure?

A

beta-blockers

21
Q

Mannitol is a useful diuretic for patients with decompensated heart failure

A

F

22
Q

Which of the following conditions is likely to precipitate symptomatic heart failure in patients with previously compensated left ventricular contractile dysfunction ?

A

All of the above

23
Q

Digoxin therapy decreases mortality rates in patients with chronic heart failure

A

F

24
Q

Pulmonary capillary wedge pressures of 13 to 17 mmHg are commonly responsible for pulmonary vascular redistribution and interstitial edema on the chest roentgenogram.

A

F

25
Q

A 72-year-old white female, previously well controlled on a once-daily combination pill containing atenolol 50 mg and hydrochlorothiazide 25 mg, presents with a rise in her blood pressure to 170/110. You add 5 mg of lisinopril, and her creatinine rises from 1.1 to 1.9. What do you suspect?

A

Atherosclerotic renal artery stenosis

26
Q

A 47-year-old male with diabetes presents as a new patient to your clinic. He does not recall any abnormal blood pressure readings. You find his blood pressure to be 138/86 on two readings during this visit. You should

A

Provide lifestyle counseling and recheck blood pressure within a few months

27
Q

A 60-year-old man presents with an acute anterior MI. He receives tPA in the emergency room one hour after onset of symptoms with rapid resolution of pain and ST elevations. He is admitted to the coronary care unit feeling well, pain free, and with stable blood pressure and pulse. Which intervention is least appropriate next?

A

Urgent cardiac catheterization with possible percutaneous coronary intervention

28
Q

Urgent cardiac catheterization with possible percutaneous coronary intervention

A

Urgent echocardiogram and surgical consultation

29
Q

A 31-year-old woman is 28 weeks pregnant with her first child. She is relatively healthy and has had an uneventful pregnancy. Her obstetrician asked her to call you because she has had increasing difficulty breathing at night. In fact she is now using three pillows and occasionally wakes up gasping for air. In your office she appears dyspneic when speaking in full sentences. Her blood pressure is 95/65 and her pulse is 118. Lung examination is notable for crackles at both bases. Cardiac exam reveals a loud S1, and an extra sound in diastole. ECG shows deep inverted P waves in V1 and a rightward QRS axis. She has 1+ pitting edema in the lower extremities. Which of the following is true?

A

The likely diagnosis is mitral stenosis, and she should respond to furosemide and â-blockers

30
Q

Hypertension resolves in almost all patients after discontinuation of oral contraceptives

A

F

31
Q

The ability to mount a reflex tachycardia

A

F

32
Q

According to the ACC/AHA Practice Guidelines for Chronic Heart Failure, which of the following statements is false?

A

None of the above

33
Q

A 70-year-old man presents with dyspnea on exertion. Physical examination reveals a blood pressure of 160/95 mm Hg and pulmonary rales at the base. An echocardiogram reveals left ventricular hypertrophy and the diastolic dysfunction. Which of the following medications would not be an initial choice?

A

Digoxin

34
Q

What is the most common etiology of tricuspid regurgitation?

A

Left-sided heart failure

35
Q

A 55-year-old white man is referred for evaluation of hypertension (BP 185/95 mm Hg), discovered during a blood pressure screening at his workplace. The patient states that he is well and has not seen a physician in many years. He describes himself as “a fitness freak,” as he is an active jogger, abstains from alcohol, and limits his salt and fat intake. He denies any knowledge of hypertension, cardiovascular disease, renal disease, or diabetes mellitus. He takes no medications regularly. Family history is significant in that his father was known to be hypertensive and died of a stroke. His older brother is being treated for hypertension. On examination, the patient appears well, with a blood pressure of 178/96 mm Hg while seated and standing. Body weight is 71 kg (157 lb), and height is 178 cm (70 in). Optic fundus examination is significant for grade II hypertensive retinopathy. The remainder of the examination is normal. Complete blood count, electrolyte panel, blood urea nitrogen level, creatinine concentration, thyroid-stimulating hormone level, and results of urinalysis are normal. Electrocardiography demonstrates normal sinus rhythm with left ventricular hypertrophy. To reduce the patient’s cardiovascular morbidity and mortality, which therapy would you prescribe?

A

Losartan